You are on page 1of 14
2017- WOFKDOOK @| Detailed Explanations of a Try Yourself Questions Electronics Engineering Analog Electronics m1. T2. wownmadeeasypublications.org (a) In this question we need to determine which diode is on and which diode is OFF. clearly diode D, is OFF because ifit is on then current from current source will low from nto p terminal of the diode D, and this is not possible, hence D, 's OFF. Applying the same concept, we can say diode D,is also OFF Diode D, is on because itis forced by the battery of 10 V. (c) Assume D,on, D,off, Dyon tov -fov nov 10-0.6-(-20) _ 4 47 my Toy = 20k =147mA Ing = 0 = 90.610) _ 5.94 ma iad 10k oe mADE Diode Applications 13, TA, f ERSY Detalled Explanation Try Yourself Questions (c) Since all the voltage are positive all the diodes willy to be forward biased but only the diode with highest voltage will be switched on rest will be in off state. (b) For negative cycle of input thus the V,, will appear across the series combination of the two 1 k@ resistors and we are taking oulput across 1 k@ resistance only hence the output will be reduced by 50 % and the above circuit will work as a full wave rectifier with an attenuation of 1/2. © Copyright ) MADE EASY Publications TS. (b) Since there is a D.C level shift in the output waveform thus the circuit must be a clamper circuit and when the diode is conducting then the voltage at the output must be 5 V as seen from the output waveform hence option (b) MADE EASY Publications MADE ERSY Heaton Workbook BJT Biasing Detalled Explanation 9 of Try Yourself Questions T1. Solution: excise = Beinn = TeyRe~ Vo = 100% 17 pA = 1.7 mA 6-15mAxR, KVL in output loop R= tov sma fon B, 150 oa Vage2sv Ven" 02 = 0.01 mA Ig, wil be equal to J, a8 there is no change in R, Ve= 2k 1.7 mA Ton = Bolen 200 x 0.01 mA = 2 mA Vow = Voo= Heo 6 -2mA x 2k = 2V = The new operating point is Q(2V, 2mA) a= 36V = 0.2V T2. Solutio 2kQx 17 mA=34V Assume Qis in active region 10-36 Tyg = 2=38 = 2.13mA ako Sea 3kKQ -5V ] a T3. Solution: =0.2mA and J, = 0.3MA. Son, = 2 2 : because we need to find minimum 5-07 number of BUT required lessne = escive - BOK FOIA 43, © 59 0 7A wowmadeeasypubliations org MADE ERSY © Copyright Small Signal Analysis of BJT Detalled Explanation of Try Yourself Questions T1. (d) 2 Oy = 2 MS jr, = 250 Ko B _ 100 = Bie= G= amg = SOKO Ww zon e| c Wr =O oni : ce 50k 50k +270 1+ gf.) (-0.994v) —(1+ 2mS x 250 kQ) x 0.994v, Viq = 497.9 y, © Copyright T2, (d) av 39k 220k \}-— ry c ol v DCetreuit - wv s9Ktit +H), ( 220k ‘ + Veet 1+ B)lg= 1.97 mA Me. 26m _igisa Ie | 197 mA wwwmadeeasypublications.org Electronics Engineering © Analog Electronics 220 k 8 aie 220k Be 220A, = 0,752k |11.578k PK Bibra Doe B ant Been = 0.509 kat = 509.4 2 = 220k V, = ~(220k 113.9%) Bly A - 83k te 73.15 = 201.44 MADE ERSY ©copyright) { wormmadeeasypublications.org 4 FET Biasing Detalled Explanation 9 of Try Yourself Questions Tt. (a) MOSFET M, tov 2 q Ip = Krag (Vigga — V5 L_ (Tae Es O.8mA = 361 5 [7 Js (2-1) ———1 oy, [i ————v, T2. (c) It Vpp= 0.4 v PMOS in depletion mode W=15V en Ad Vp = Vq «. we conclude that each MOSFET isin saturation Ip = Ky Wag~ Va? MOSFET M, ky Wass— Vi)? Vas = 10-5=5v Vp > Vgq + Vr current saturation L(w : V9 < Ve + Vn4 Triode rogion w Vag = Vg=Vp=15-0=15v (4) =1738 1.5 >1V-4 04 cuirént saturation region vs=09V Vp=09V MOSFET M, oav osv 0.5 my We (7) t}, = 694 (Ceara MADE ERSY my wwwmadeeasypublications.org 8 Electronics Engineering * Analog Electronics MADE EASY Publieatone PMOS in depletion mode. M, & M, are in current saturation Vay = V5~-Vp=0.9-0.9=0 3— Vos; ~ Voge = 9 Veg = Vg-Vq = 09-0 =0.9 0<0.9+04 triode region 3. (b) 93v hm 4 —™ Given Vyy= tv, So MOSFET is an n channel enhancement MOSFET in both transistors Yp= Va wowmadeeasypublications org MADE EASY Publications 1 w 2 Fin Cox Pasi) Px20xs(1 5-1 uA =75 HA Small Signal Analysis of FET Detalled Explanation of Try Yourself Questions TH. (b) Itis common drain amplifier, R. 4ko aw anh _Gm4K__ gg Teg Re T+ gy 4KQ py = 4.75 Ip = 2 Ky (Veg Vp) wa (fo) py = 2 Ky 1 w gp = 2 0% Zin Cox (4) w wag uomn e copyright MADE ERSY wwwmadeeasypublications.org Multistage Amplifiers Detalled Explanation 9 of Try Yourself Questions TH. (b) The input resistance will be Ri = 1, + (B, + 1)Re = Tk + (101)(1K) = 102 ka + (hy + DR = Tk (51) (102k) = 5.203 Ma wwrwimadeeasypublications.org MADE ERSY © Copyright Op-Amps and 555 Timer Detalled Explanation 9 of Try Yourself Questions Tt. (b) voltage shunt Output of op-amp 1 It is connected to schmitt trigger (inverting My. mode) — clockwise B=y-4 But inverting ampli + inverting schmit tigger = anticlockwise. pee “Vy ~ 70k __ to tox! 12. (b) 10k toxtox 10° pete Ko ona 2ka 13. () Wr > oy Redrawing the circuit by replacing amplifier with LQ £ its block clagram from the given properties Zoo A, =<; Ry = 0; voltage gain = Ay © Copyright MADE EASY www madeeasypublications org 12 Electronics Engineering © Analog Electronics R, in We Ave Publications From the circuit 1,8, V, Virtual short concept) It we apply KVL between node Band C, Vp = Vo(Virtual short concept) Vio lip= and = 15. (a) The duty cycle of the above astable muitivibrator (designed! using 555 timer) is Ton, T Rat Re . Fests thus Duty cyclo > 60% —————e (Wrwwimadeeasypublications.org mADE EASY © Copyright | Negative Feedback Amplifiers Detalled Explanation of Try Yourself Questions T1 (a) 2. (b) The overall forward gain is 1000 and close loop The feedback element is Fit samples voltage gain is 100. Thus, f= 0.009 and mix current so shunt-shunt feedback Now, when gain of each stage increase by 10% then overall forward gain will be 1831 and using the previous value of B the close loop will be 102.55. = Close loop Voltage gain increase by 2.55%. © Copyright MADE EASY www madeeasypublications org @ Detalled Explanation of Try Yourself Questions TH. (a) 72. (a) The output can be +12 V only Since their are 3 capacitors the maximum when outputis 12 V then phase shift that can be provided will be 270° R, but due to the presence of the RC circuit the 8 phase shift is equal to 60° for the individual c he RC circuit, making the phase shift of the ot feedback network equal to 180°. Thus the = amplifier should be an inverting ampitier so i that it can be a positive feedback circuit and 2ia because the amplifier is a practical amplifier P. po thus [ABI> 1 for the circuit to work. 10Ka 10Ka s v,=6V when output is -12 Vthen 10K wwrwimadeeasypublications.org MADE ERSY © Copyright

You might also like